General ACT and SAT Test-Taking Tips

  1. Schedule your test well in advance, and try to select a site that is connected to a high school. Sites that are not connected to high schools have less accountability to the students and tend to be more likely to cancel tests at the last moment. Taking the test at your high school is generally best. If your school isn’t a testing center, consider asking the administration why that is.
  2. Make sure to read your emails! The ACT and SAT will both email you occasionally with important information. It might be a reminder to upload your picture (which you can’t take the ACT without). It might be an update to your testing location (so you show up at the right place). It might be the notification to set up your SAT prior to arriving at your test center (you can’t do it once you arrive). Many students ignore these emails and then have big problems on test day.
  3. Have a regular schedule for practice and tutoring. Having a scheduled time in your week to practice or meet with a tutor will provide better results than intermittent or random practice squeezed into any free spot.
  4. Practice like you play. When doing practice at home, make sure you are giving it your all and treating it like test day. Practicing while lounging on the couch, eating a snack, and intermittently texting a friend will be far less effective than sitting at a desk or table, timing yourself, and focusing without interruption.
  5. Practice what you’re good at too. A lot of students make the mistake of only focusing on their weaknesses. It is often easy to improve what you’re already good at, so don’t forget to put effort into each section of your test.
  6. Don’t cut yourself slack. It’s very tempting to ignore small mistakes. “Oh, it was just a silly slip up” is easy to say. Small mistakes lead to wrong answers, just like big mistakes do, so treat a small mistake just as seriously as a big one. Identify why that small mistake happened and work towards eliminating what caused it.
  7. Sleep. High school students generally need over 8 hours of sleep each night. Just because you can exist on less doesn’t mean that is ideal. Best brain function occurs when students are well rested the week or two prior to the test. Start practicing healthy sleep habits today.

Digital SAT Practice Questions–Command of Evidence: Quantitative

Here are 10 original questions to practice the new Digital SAT Quantitative Command of Evidence questions.

  1. Consumption of Sugar, Coffee, and Tea:

CountrySugar (lbs)Coffee (lbs)Tea (lbs)
Great Britain35.960.903.190
United States24.635.68—–
Holland14.867.030.800
France14.392.320.018
Norway11.046.920.060
Sweden9.800.800.060
Switzerland9.605.28—-
Germany9.424.030.035
Denmark9.003.400.400
Belgium7.188.590.018
Portugal6.330.690.040
Italy5.200.900.020
Austria4.931.300.012
Spain4.230.010.040
Russia2.400.0070.160

The entire consumption of sugar in Europe has averaged, during the last few years, 3,410,000 pounds, and for the whole world is it set down at nearly twice that amount. It is estimated that three fourths of the sugar is made from cane, and one fourth from beet. The consumption of coffee has doubled in most countries during the last twenty years.


A scientist wishes to use data from the table to try to back up the initial claim of the author as to European sugar consumption. Which of the following procedures should the scientist undertake in order to utilize the data?

A. Add up all the sugar from European countries and see if it is roughly 3,410,000 pounds.
B. Add up all the sugar from all countries and see if it is exactly 3,410,000 pounds
C. Add up all the coffee from the European countries and see if it is double 3,410,000 pounds
D. Add up all the coffee from the all the countries and see if it is exactly 3,410,000 pounds.

2. Annual report to Congress of the Commissioner of Patents, 1876:

Number of applications for patents in 187621,425
Number of patents issued, including reissues and designs15,595
Number of applications for extension of patents2
Number of patents extended3
Number of caveats filed during the year2,697
Number of patents expired during the year814
Number of patents allowed but not issued for want of final fee3,353
Number of applications for registering of trademarks1,081
Number of trademarks registered959
Number of applications for registering of labels650
Number of labels registered402

The number of applications for patents was a little less than during the previous year. The Commissioner suggests that Congress should appropriate $50,000 to promote the printing of the old patents; that additional examiners be employed, and more clerks, for the purpose of expiating the business of the office; that the price of the Official Gazette be reduced, also the fee for trademark registration; that the library fund be increased; that more space be provided for models, and the transaction of business.


The author is requesting additional funds to hire employees for the patent office. Which answer, if true, would support his claim that more examiners and clerks are needed?

A. Of the 15,595 patents issued, most were to citizens of the United States.
B. Of the roughly 21,425 applications, over half could not be reviewed in time for the annual report.
C. Three patent extensions were needed in 1876
D. The 3,353 patents not issued for lack of fee were mostly to non-Americans



3.

 Without ConductorWith Conductor
Moisture %78.2179.84
Sugar16.8618.41
Tartaric acid0.8000.791
Bitartrate of potash0.1800.186

Macagno, also believing that the passage of electricity from air through the vine to earth would stimulate growth, selected a certain number of vines, all of the same variety and all in the same condition of health and development. Sixteen vines were submitted to experiment and sixteen were left to natural influences. In the ends of the vines under treatment, pointed platinum wires were inserted, to which were attached copper wires, leading to the tops of tall poles near the vines; at the base of these same vines other platinum wires were inserted and connected by copper wires with the soil. At the close of the experiment, the wood, leaves, and fruit of both sets of vines were submitted to careful analysis with the above results.

Which answer would be the best summary statement to make based on the results in the chart as they relate to Mocagno’s hypothesis?

A. The plants without the conductor and the plants with the conductor had no measurable difference in bitartrate of potash.
B. The plants with the conductor had higher levels of moisture, sugar, and bitartrate of potash, which created growth and proves Macagno’s hypothesis correct.
C. Because the plants without a conductor had higher tartaric acid levels, they would have grown more than the plans with the conductor, proving Macagno’s hypothesis incorrect.
D. Because we do not have data on the size or height of the plants, we can not draw conclusions as to the effect of the conductor on plant growth. Macagno’s hypothesis remains untested.

4. The following table gives the absolute sensitiveness of several of the best known kinds of American and foreign photography plates, when developed with oxalate, in terms of pure silver chloride taken as a standard. As the numbers would be very large, however, if the chloride were taken as a unit, it was thought better to give them in even hundred thousands.

Sensitiveness of Plates:

PlatesDaylightGaslight
Carbutt transparency0.7
Allen and Rowell1.3150
Richardson Standard1.310
Marchall and Blair2.7140
Blair Instantaneous3.0140
Carbutt Special4.020
Monroe4.025
Wratten and Wainwright4.010
Eastman special5.330
Richardson Instantaneous5.320
Walker Reid and Inglis11.0600
Edwards11.020
Monckhoven16.0120
Beebe16.020
Cramer16.0120

It will be noted that the plates most sensitive to gaslight are by no means necessarily the most sensitive to daylight; in some instances, in fact,                                                                . It should be said that the above figures cannot be considered final until each plate has been tested separately with its own developer, as this would undoubtedly have some influence on the final result.

Which answer option best completes the passage with information consistent with the data in the table?


A. there is no gaslight sensitivity whatsoever
B. the gaslight and daylight sensitivity seem to be nearly the same
C. the reverse seems to be true
D. The daylight sensitivity is far below the gaslight sensitivity

5. So much has been claimed for natural gas as regards the superiority of its heating properties as compared with coal, that some analyses of this gas, together with calculations showing the comparison between its heating power and that of coal, may be of interest. These calculations are, of course, theoretical in both cases, and it must not be imagined that the total amount of heat, either in a ton of coal or 1,000 cubic feet of natural gas, can ever be fully utilized. In making these calculations I employed as a basis what in my estimation was a gas of an average chemical composition, as I have found that gas from the same well caries continually in its compositions. Thus, samples of gas from the same well, but taken on different days, _______________________________________  and so with all the component gases.

Analysis of Natural Gas- given as percents

Date tested10/28/8410/29/8411/24/8412/4/8410/18/8410/25/84
Carbonic Acid0.8.60.400.3
Carbonic Oxide1.00.8.580.41.00.30
Oxygen1.10.8.780.82.11.2
Olefiant Gas0.70.80.980.60.80.6
Ethylic Hydride3.65.57.9212.305.24.8
March Gas72.1865.2560.7049.5857.8575.16
Hydrogen20.0226.1629.0335.929.6414.45
Nitrogen000023.412.89
Heat Units728,746698,170627,170745,813592,380745,591

Which answer option best completes the passage with information from the chart?


A. vary in olefiant gas from .98 to .6, Ethylic Hydride from 12.3 to 3.6, heat units from 745,813 to 592,380,
B. vary in nitrogen from 23 percent to zero percent, carbonic acid from two percent to zero percent, oxygen from four percent to 0.4 percent,
C. vary in nitrogen from 23 percent to zero percent, carbonic acid from two percent to zero percent, heat units from 745,813 to 592,380,
D. vary in nitrogen from 23.41 to 0, carbonic acid from .08 to .3, Oxygen from 1.1 to 1.2

6. The following table gives some particulars of the Great Lakes and the discharge from them:

LakeElevation above mean tideArea of Basin (square miles)Area of lake (square miles)RainfallEvaporationDischarge
Superior602.7890,50538.875187,38634,49580,870
Huron and Michigan581.28121,94150,400262,96466,754216,435
Erie572.8640,29810,00096,65413,870234,578
Ontario246.6131,5587,22075,69210,568272,095

The average variation in level of the lakes is from 18 inches to 24 inches during the year, and the range in evaporation from year to year is also very considerable; thus the evaporation per second on Huron and Michigan, as given in the table above, _____________________  but the figures for another year show nearly 89,000 feet per second, which would represent a difference of 6.5 inches in water level. As a discharge of 10,000 cubic feet a second into the new canal would lower the level of these two lakes by 2.87 inches in a year, it follows that the difference between a year of maximum and one of minimum evaporations is more than twice as great as would be required for the canal, and even under the most unfavorable conditions the volume taken from the whole chain of lakes would not lower them an inch.

Which answer option best completes the passage with relevant data from the table?


A. is nearly 67,000 feet,
B. is around 250,000 feet,
C. is nearly 14,000 feet,
D. is nearly 35,000 feet,

7. The portion of the flame which is supposed to be the hottest is about half an inch above the tip of the inner zone of the flame, and it is at this point that most vessels containing water to be heated are made to impinge on the flame; and it is this portion of the flame, also, which is utilized for raising various solids to a temperature at which they radiate heat.

In order to gain an insight into the amount of contamination which the air undergoes when a geyser or cooking stove is at work, I have determined the composition of the products of a combustion, and the unburned gases escaping when a vessel containing water at the ordinary temperatures is heated up to the boiling point by a gas flame, the vessel being placed, in the fist case, half an inch above the inner cone of the flame, and in the second, at the extreme outer tip of the flame.

Gases Escaping During Combustion:

 Luminous flame InnerLuminous flame Outer
Nitrogen77.5269.41
Water Vapor11.8019.24
Carbon Dioxide4.938.38
Carbon Monoxide2.452.58
March Gas0.950.39
Acetylene0.270
Hydrogen2.080
   
 100.00100.00


Based on the passage and the table, what inference could be made about the placement of the water vessel and the resulting effects on the air in the room the experiment was conducted?

A.  When water was heated in a vessel placed in the hottest part of the flame, less of every type of contaminant was released into the air, leading to better air quality than when the vessel was placed in a cooler part of the flame.
B. When water was heated in a vessel placed in the hottest part of the flame, more nitrogen, march gas, acetylene, and hydrogen were released into the air while less water vapor, carbon monoxide, and carbon dioxide were released. The resulting effect on air quality is unknown.
C. When water was heated in a vessel placed in the hottest part of the flame, less nitrogen, march gas acetylene, and hydrogen were released into the air while more water vapor, carbon monoxide, and carbon dioxide were released. The resulting effect on air quality is positive.
D. When water was heated in a vessel placed in the hottest part of the flame, more of every type of contaminant was released into the air, leading to poorer air quality than when the vessel was placed in a cooler part of the flame.



8. Eggs Laid and Gain in Weight in Hens over the course of three periods.

 NitrogenousCarbonaceous
Live Weight, July 26th23.5323.56
Live weight, November 2721.3122.00
Loss2.221.56
Number of Eggs laid79.0026.00
Weight of eggs laid (lb)8.252.92
Average weight of eggs (oz)1.671.80
Gain in weight, including eggs (lb)6.031.36

During the first week the carbonaceous fed hens laid three eggs while the others laid two. The two groups were, therefore, practically evenly divided at the start as to the condition of the laying stage. At the end of the first period the nitrogenous fed hens had laid forty-three eggs and the carbonaceous fed hens had laid twenty. During the next twenty-five days the former laid thirty and the latter six; ___________________________________     From this time on no eggs were received from either group. The decline in egg production was probably due in large part to the fact that the hens began to molt during the second period, and continued to do so during the rest of the experiment.

Which answer option best uses information from the table to complete the passage?


A. during the third period the former laid six and the latter not any.
B. during the third period the former laid 79 and the later laid 26.
C. during the third period the former laid non and the latter laid six.
D. during the third period it was no possible to tabulate the number of eggs laid.



9. Cost of living for a man in Great Britain and the United States, in shillings:

 Great BritianUnited States
Food744773.7
Clothing192278.4
Rent144272.1
Fuel60122.0
Sundries6090.0
Total:1200 Shillings
(60 pounds)
1536.2 Shillings (77 pounds, 16 shillings)

Having agreed that wages are probably 62 percent higher in Massachusetts than in Great Britain, it would be easy, if we could ascertain what proportion of a working man’s income is spent respectively in groceries, provisions, clothing etc., to determine what advantage an operative derives from the higher wages of the United States. Dr. Engle, the chief of the Prussian Bureau of Statistics, puts us in possession of this information, and, as the result of a laborious inquiry, has formulated a certain economic law which governs the relations between income and expenditure. We learn, consequently ___________________________ .

Which answer best uses information from the table and the passage to draw a conclusion about wage equity between the United States and Great Britain?

A. that a workman earning 1200 shillings per year in Great Britain would also have to pay more in food costs, thus further reducing his available funds, and making the advantage of working in the United States even stronger. 
B. that workers in the United States have to pay more for everything from food to Sundries, making their income lower than that of a man in a comparable job in Great Britain.
C. that a workman in Great Britain earns only 60 pounds per year while a comparable workman in the United States earns over 77 pounds, creating a wealth gap of 17 pounds, 16 shillings between English and American workers.
D. that a workman earning 60 pounds per annum in Great Britain would receive 99 pounds in the States, but living there would cost him 77 pounds, or 17 pounds more than here, giving him a net advantage of only 22 pounds.

10. The following table, which has been prepared by the French Ministry of Public Works, gives the railway mileage of the various countries of Europe and the United States up to the end of 1881, with the number of miles constructed in that year, and the population per mile:

CountryTotal MilesMiles Built in 1881Population per Mile
Germany21,3133312,154
Great Britain18,1571641,939
France17,1348952,170
Austria-Hungary11,8802623,200
Italy5,4501095,321
Spain4,8691763,492
Sweden and Norway4,6162731,408
Belgium2,561482,203
Switzerland1,557221,831
Holland1,425832,885
Denmark1,053251,919
Romania916565,860
Turkey8662,891
Portugal75785,870
Greece628,000
Total107,3062,4553,168
United States104,8139,358502

It appears from this that the United States Mileage was only 2,493 less than the total of all Europe, and at the present time it exceeds it, as the former country has built about 6,000 miles this year, whereas Europe has not exceeded 1,500. The difference in the number of persons per mile in the two cases is also

Which answer, if true, would best complete the author’s thought, using data from the table?


A. generally minimal, with the difference between countries varying from 5,870 in Portugal to 1,405 in Sweden and Norway with two outliers: the United States and Greece.
B. very great: Greece has one mile of rail for every 28,000 people while Sweden and Norway have one mile of rail for every 1,408 people largely due to the density of their population.
C. very great, Europe taking six times as many persons to support a mile of railway as the States, and can only be accounted for by the fact that American railways are constructed much cheaper than the European ones.
D. marginal, with the average population per mile in Europe hovering around 4,750.




Answer Solutions

1.  A. The passage claims that Europe has, in the past few years, had a sugar consumption around 3,410,000 pounds. If a scientist wanted to support this, he or she could find the total European sugar consumption for the year in the table and see if it is close to the average. Of course, any given year might be an outlier, but given the limited data in the table, this is still the best option. Option B is incorrect because it includes non-European countries. Options C and D are correct because the scientist is trying to make a claim about sugar, not coffee.
2. B. In the passage, the commissioner requests additional funds in order to employ more examiners and clerks for the patent office “for the purpose of expiating the business of the office”. In other words, they need more people so that the office can move more quickly in processing the patent and other applications. If it were true that only half of the submitted patents had been able to be reviewed by the time the end of the year report came out, that would support the idea that more workers are needed, making option B correct. Option A is incorrect, as the citizenship of the applicants does not impact the need for more funds. Options C and D are incorrect for the same reason.
3. D. The hypothesis posited by Mscagno was that electricity would stimulate the growth of the plants. Because plant height, weight, or other indicators of growth were not measured, the data cannot be used to support Macagno’s hypothesis. This makes option D correct and the other answers incorrect. The presence of moisture, sugar, tartaric acid, and bitartrate of potash are not indicators of growth.
4. C. The first sentence points out that there seems to be no positive correlation between plate sensitivity to daylight and gaslight. The passage continues by saying that “in some instances, in fact”. This is leading into a contrasting statement from the first sentence, making option C the best answer. Answer option A is only correct for Carbutt transparencies, not for the plates in general. Answer B is incorrect as there is a clear and measurable difference between daylight and gaslight sensitivity for all the plates. Option D is incorrect as it does not contrast with the first sentence of the passage.
5. B. The correct answer must include the variances for different components of the gas. Heat units are not components of gas, but rather a measure of energy, and thus answers A, and C are incorrect. Answer option D incorrectly measures the highest and lowest measurements of the components, leaving only answer B as a correct option.
6. A. A careful reading of the graph shows that the evaporation on lakes Huron and Michigan is 66,754 which makes option A correct and the other options incorrect.
7. B. We learn in the first paragraph that the hottest part of the flame is about half an inch above the tip of the inner zone of the flame. This makes “luminous flame inner” our hotter option and “luminous flame outer” our cooler option. Based on this, we can see that the hotter option has higher outputs of nitrogen, march gas, acetylene, and hydrogen and lower outputs of water vapor, carbon monoxide, and carbon dioxide. However, we do not have evidence of the impact of these outputs on air quality. This makes option B the best answer. The other answer options are all incorrect as they misunderstand either which one is hotter, or they assume the impact on the air quality without evidence.
8. A. We learn in the table that the nitrogenous fed (former) hens laid a total of 79 eggs and the carbonaceous fed (later) hens laid a total of 26.  The passage tells us that in the first and second periods the nitrogenous fed hens laid 43 and 25 eggs and the carbonaceous fed hens laid 20 and 6. 79-43-25=6 therefore in the third period the nitrogenous fed hens laid six eggs. 26-20-6=0 therefore in the third period the carbonaceous fed hens laid 0. This is best stated in answer option A.
9. D. We see in the table that in Great Britain the cost of living is 1200 shillings, or 60 pounds. In the United States, the cost of living is 1536.2 shillings, or 77 pounds, 16 shillings. We learn in the passage that wages are higher in Massachusetts than in Great Britain. This makes answer D the most logical conclusion. A that pays 60 pounds in Great Britain would likely pay more in the United States, but some of that advantage would be eaten away by the higher cost of living.
10. C.  Note the word “also” before the blank. In the previous sentence, the author pointed out a large discrepancy in the 6000 miles built in the U.S. compared to the only 1500 built in Europe. We now need a similar comparison for that “also” to make sense. This would mean that answers A and D are incorrect as they do not show a similarly large difference. Answer B is incorrect as it compares one European country to another instead of continuing the author’s comparison of all of Europe to the United States.





Sources:

All information and data are taken from or adapted from various editions of Scientific American and the supplements thereof. You can improve your reading skills by reading more of similar texts. Specific links for the information from each question can be found below.

1. https://www.gutenberg.org/cache/epub/19180/pg19180-images.html
2. https://www.gutenberg.org/cache/epub/19406/pg19406-images.html
3. https://www.gutenberg.org/cache/epub/15193/pg15193-images.html
4. https://www.gutenberg.org/cache/epub/13401/pg13401-images.html
5. https://www.gutenberg.org/cache/epub/13401/pg13401-images.html
6. https://www.gutenberg.org/cache/epub/13443/pg13443-images.html
https://www.gutenberg.org/cache/epub/13443/pg13443-images.html
8. https://www.gutenberg.org/cache/epub/13443/pg13443-images.html
9. https://www.gutenberg.org/cache/epub/11735/pg11735-images.html
10. https://www.gutenberg.org/cache/epub/8504/pg8504-images.html

Yale will again require standardized test scores

Yale just announced that they will again require test scores as part of the college application. In addition to SAT or ACT scores, students can now submit Advanced Placement or International Baccalaureate results. Yale made this shift because it will improve their ability to predict which students are most likely to succeed at Yale:

“Yale’s research from before and after the pandemic has consistently demonstrated that, among all application components, test scores are the single greatest predictor of a student’s future Yale grades. This is true even after controlling for family income and other demographic variables, and it is true for subject-based exams such as AP and IB, in addition to the ACT and SAT.”

In addition, Yale found that including standardized test scores would serve to increase the diversity of its class:

“Our researchers and readers found that when admissions officers reviewed applications with no scores, they placed greater weight on other parts of the application. But this shift frequently worked to the disadvantage of applicants from lower socio-economic backgrounds.” Source: https://admissions.yale.edu/test-flexible

Students looking to apply to highly selective schools would be well-served by showing their readiness for college level work by doing their best on the SAT or ACT.

Is the Digital SAT easier than the old paper SAT?

Many students and parents who were familiar with the old paper-based SAT are wondering whether the new Digital SAT will be easier.  The Digital SAT is designed to maintain the same level of rigor and predictive value for colleges as the paper SAT—otherwise, it wouldn’t be a useful tool to assess freshman-year readiness.  The College Board has done extensive research to ensure that Digital SAT scores align with paper SAT scores.  That being said, students I have tutored have generally found the new Digital SAT to be much less intimidating and more approachable than the paper SAT.  Here are ten reasons why most students will find the Digital SAT easier than the paper SAT. 

1.  Test fatigue is much less of an issue.  While the paper SAT was over three hours long, the Digital SAT is only a little over two hours.  This is possible because of the section-adaptive format of the Digital SAT:  students start with modules of average difficulty and then progress to either more or less challenging modules based on their first module performance.  Most students find that sustaining their attention for a little over two hours is quite manageable, making the Digital SAT less overwhelming than the paper SAT. 

2.  Less time wasted on double-checking.  On the paper SAT, many students found it difficult to avoid looking back at their previous answers since they could easily be seen.  Also, they had concerns about their bubbling in of the paper answer sheets. On the Digital SAT, students will only view one question at a time, making it much easier to compartmentalize their focus on one task.  Also, students click on the answer instead of physically bubbling, and they can easily see that the answer choice they selected is recorded by the computer. 

3.  Calculators are available throughout the math section.  The paper SAT had a no-calculator section; even though all the problems on this section could have been done without a calculator, many students found it more challenging than the calculator math section.  On the Digital SAT, students can bring a calculator of their own to use for the math sections.  Moreover, they have access to the powerful Desmos™ calculator that is built into the testing interface.  The functionality of the Desmos™ calculator allows students to easily graph parabolas, systems of equations, and even tables. 

4.  Fewer questions on obscure grammar concepts.  The Digital SAT focuses on grammar fundamentals:  subject-verb agreement, verb tense, punctuation, modifier placement, and transitions.  Unlike the old SAT, which also tested idioms, diction, and wordiness, students will find that they can focus their grammar study on certain concepts.  For example, understanding the rules of semicolon and colon usage can go a long way on the Digital SAT. 

5. The question stems are more predictable.  Unlike the paper SAT, which had a wide variety of question wording, the questions on the Digital SAT are quite consistent.  Students will find certain question stems repeated over and over: 

Which choice completes the text with the most logical and precise word or phrase?  

Which choice best states the main purpose of the text?

Which choice most logically completes the text? 

Which choice completes the text so that it conforms to the conventions of Standard English?  Which choice completes the text with the most logical transition?  

Having these consistent question stems allows students to get in a better testing rhythm and devote more time to thinking about their answers instead of trying to understand what they are asked to do. 

6.  The reading and writing selections are concise.  The old paper SAT had just a few reading and writing passages, each of which was several hundred words.  If a student found a topic uninteresting on these sorts of passages, it was easy for them to lose focus.  On the Digital SAT, the reading and writing passages are no longer than 150 words and are accompanied by a single question.  While a shorter text doesn’t necessarily mean an easier text, it does mean that students will often find them less daunting than the longer old SAT passages. 

7.  Proctor errors are less of a concern.  I have had some past tutoring students experience proctor errors during their testing—in particular, the proctor called “time” too early or too late.  Since the timing on the Digital SAT is done within the testing application, students need not worry about time being called early.  If they hide the countdown clock, it will automatically reappear when 5 minutes remain.  That way, students can be sure they answer every question before they run out of time. 

8. Students can use their own tablet or computer.  Familiarity brings comfort—students can practice on the same tablet or computer they want to use on test day.  Students can take control of their testing process by ensuring their computer is fully charged and in good working order.  There should be a minimal disconnect between practice tests and real tests when the device is the same for each. 

9.  The questions are less wordy.  Both the reading/writing and the math questions on the Digital SAT are generally shorter than those found on the paper SAT.  If students take their time carefully reading the questions, they are unlikely to make careless reading errors. 

10.  The Digital SAT is almost exactly like the Digital PSAT.  While there are slight differences in the content tested on the SAT and PSAT, the two tests have the same format and time restrictions.  Students who took the PSAT in the fall will feel comfortable knowing that they have already seen the computer interface and question types they will see on the Digital SAT. 

All told, students should look at the new Digital SAT not as something to be feared, but as an outstanding opportunity to demonstrate their academic skills to colleges. 

Digital SAT Writing Transitions Practice Questions

1. The following is an excerpt from The Dorrington Deed-box by Arthur Morrison:


As for Dorrington, he had his hundred pounds reward. But the bill for £10,000 he never presented. Why, I do not altogether know, unless he found that Mr. Mallows’s financial position, as he had hinted, was not altogether so good as was supposed.  __________ it was found among the notes and telegrams in this case in the Dorrington deed-box.

Which choice completes the text with the most logical and precise word or phrase?

A. In other words,
B. At any rate,
C. Whatever,
D. On the other hand,

2. The following is an excerpt from Sanders of the River, By Edgar Wallace:


The Hon. George Tackle had the good fortune to be the son of his father; otherwise, I am free to confess he had no claim to distinction.  __________ his father, being the proprietor of the Courier and Echo (with which are incorporated I don’t know how many dead and gone stars of the Fleet Street firmament), George had a “pull” which no amount of competitive merit could hope to contend with.

Which choice completes the text with the most logical and precise word or phrase?

A. But
B. While
C. So
D. Thus

3. The following is an excerpt from The Pathless Trail by Arthur O. Friel:


Sleep enveloped the huts. Snores and gurgles rose and fell. Tim himself, for the sake of effect, snored heartily at intervals, __________ his eyes never closed. Through his mosquito bar he could see only vaguely, but he knew any man walking from the crew’s quarters must cast a very visible shadow across that net, and to him the shadow would be as good a warning as a clear view of the substance. But the hours crept on, and no shadow came.

Which choice completes the text with the most logical and precise word or phrase?

A. and
B. though
C. since
D. for

4. The Following is an excerpt from Bones in London, By Edgar Wallace:


The Tibbetts-Jelf Lamp was something new in motor lamps. It was a lamp which had all the advantages of the old lamp, plus properties which no lamp had ever had before, and it had none of the disadvantages of any lamp previously introduced, and, __________ had no disadvantages whatsoever. So Jelf told Bones with great earnestness.

Which choice completes the text with the most logical and precise word or phrase?

A. however,
B. finally,
C. in fact,
D. therefore,

5. The following is an excerpt from The Keepers of The King’s Peace by Edgar Wallace:


The Wiggle, moreover, possessed many attributes which are denied to other small steamers. She had, __________  a Maxim gun on her tiny forecastle. She had a siren of unusual power and diabolical tone, she was also fitted with a big motor-horn, both of which appendages were Bones’s gift to his flagship.

Which choice completes the text with the most logical and precise word or phrase?


A. on the other hand,
B. however,
C. completely,
D. for example

6. The following is an excerpt from Kidnapped by Robert Lois Stevenson:

I was abashed how to find expression for my thanks; but she was no less abashed at the thought of hearing them; begged us to lose no time and to hold our peace, saying (very properly) that the heart of our matter was in haste and silence; __________ what with one thing and another, she had set us on the Lothian shore not far from Carriden, had shaken hands with us, and was out again at sea and rowing for Limekilns, before there was one word said either of her service or our gratitude.

Which choice completes the text with the most logical and precise word or phrase?


A. therefore,
B. because,
C. but,
D. and,

7. The following is an excerpt from Caves of Terror by Talbot Mundy


The tiny portions that melted and liquefied became full of motion, __________ the motion was never in one place for more than about a minute at a time; and wherever the motion had been the lump lost bulk, so that gradually the whole piece shrank and shrank.

Which choice completes the text with the most logical and precise word or phrase?

A. while
B. although
C. so
D. because

8. The following is an excerpt from The Prisoner of Zenda by Anthony Hope:

After that we called on Bertram Bertrand, a versifier of some repute and Paris correspondent to The Critic. He had a very comfortable suite of rooms, and we found some pleasant fellows smoking and talking. It struck me, __________ that Bertram himself was absent and in low spirits, and when everybody except ourselves had gone, I rallied him on his moping preoccupation.

Which choice completes the text with the most logical and precise word or phrase?

A. however,
B. on one hand,
C. therefore,
D. nevertheless,

9. The following is an excerpt from the introduction to She, by H. Rider Haggard:

That same evening my visit came to an end, and this was the last I saw or heard of “Charon” and “the Greek god” for many a long day. __________ I have never seen either of them from that hour to this, and do not think it probable that I shall. But a month ago I received a letter and two packets, one of manuscript, and on opening the first found that it was signed by “Horace Holly,” a name that at the moment was not familiar to me.

Which choice completes the text with the most logical and precise word or phrase?

A. While,
B. Therefore,
C. Indeed,
D. Nevertheless,

10. The following is an excerpt from The Lion of Petra by Talbot Mundy:

The beginning as concerns me was when I moved into quarters in Grim’s mess in Jerusalem. As a civilian and a foreigner I could not have done that, __________ if it had been a real mess; but Grim, who gets fun out of side-stepping all regulations, had established a sort of semi-military boarding-house for junior officers who were tired of tents, and he was too high up in the Intelligence Department for anybody less than the administrator to interfere with him openly.

Which choice completes the text with the most logical and precise word or phrase?

A. but,
B. in addition,
C. while,
D. of course,

Solutions

1. B.  The information before the blank is saying that the author doesn’t exactly know why Dorrington never cashed the 10,000-pound check. The sentence with the blank is saying that the check was found in his possession. The author is not repeating previous information, so answer A is incorrect. Answer C makes no logical sense. Answer D is incorrect because the author is not contrasting the two sentences. Instead, the author is offering more information about the check, making option B the best answer.

2. A. The general meaning of this passage is that George Tackle had no reason to be notable, except that his father was notable. This makes option A the best answer since we are contrasting the lack of importance of the son with the “pull” he gets from his father. Option B is incorrect as it makes the second sentence a fragment. Options C and D are incorrect as the author is not showing cause and effect. 

3. B. Here, the author is setting up a contrast. We learn that Tim is making snoring noises, but he is still watching through the mosquito netting for a shadow. Therefore, he isn’t actually asleep. The only answer option that shows the contrast between the snoring noises and his being awake is option B.

4. C. Notice the word “and” before the blank. This eliminates option A. Option B is incorrect as we are not placing things in order. Option D is incorrect as the author is not concluding. The author is offering additional information, making option C the best answer.

5. D. In this passage, the second sentence is showing an example of an attribute The Wiggle had which other small steamers did not have. This makes option D the only appropriate answer.

6. D. The author is offering more information here. The woman has refused thanks and set them on the shore. This makes option D the best answer. The author is not showing cause and effect, making A and B incorrect and is not showing contrast, making option C incorrect.

7. B. The sentence shows that the portions are full of motion. The author wants to contrast this with the motion never being regular or in one place. This contrast is best shown in answer B. Answer A leaves the sentence a fragment. Options C and D do not show contrast.

8. A. The context clue here is that they found “pleasant fellows” but Bertram was “in low spirits” these are two contrasting emotions, making “however” the best answer. Option B would need to be placed with the first item of contrast, not the second. Option C shows cause and effect, not contrast. Option D does not fit into the context of the sentence.

9. C. In this sentence, the author is adding more information to emphasize the information in the first sentence that “this was the last I saw or heard of [them] for many a long day”. This makes option C the best answer as it shows that what is coming next is additional information. Option A does not fit into the sentence structure of the second sentence. Option B shows cause and effect. Option D shows contrast.

10. D. The first sentence sets up that he could not do what he did. The second sentence explains why he could do that after all (by breaking rules). The keeping of the rules is to be assumed, thus “of course” is the best answer. The breaking of rules is not to be assumed. Options A, B, and C do not fit into the structure of the sentence.

Digital SAT Rhetorical Synthesis Practice Questions

1. While researching a topic a student has taken the following notes:

  • In Rome, there still remain ruins of the Temple of Apollo Palatinus which was constructed in the first century BCE.
  • In the late 1st century CE the temple underwent a restoration after being fire damaged in the Great Fire of Rome in 64 CE
  • The temple was almost completely destroyed in another fire in 363 CE
  • If you visit Rome today you will only be able to see the core of the temple’s podium and some other fragments which were excavated in the mid-1800s.

The student wants to educate visitors to Rome as to why they can’t see the entire temple today. Which choice most effectively uses relevant information from the notes to accomplish this goal?

A. Because of an excavation in the 1800s, parts of the temple including the podium, are visible to modern visitors to the site.
B. After the Great Fire of Rome in 64 CE the entire temple underwent a restoration so that it could continue being used.
C. The Temple of Apollo Palatinus was originally built over 2000 years ago.
D. After its nearly complete destruction in a fire in 363, the Temple of Apollo Palatinus was not restored, leaving only fragments.

2. While researching a topic a student has taken the following notes:

  • The Fountain and Tallman Museum is located in the historic Fountain-Tallman Soda Works building in Placerville California.
  • The building is unique in its construction as it has stone walls that are over two feet thick—originally designed to keep ice and other soda making equipment cool. The thick walls are why it still stands when most other buildings from that era have not survived.
  • In addition to being a soda water factory, the building was also used as a jail and an office space for a gas company.
  • In the late 1900s the building was donated to a local historical society and was set up as a museum of local history.

The student wants to introduce the historic Fountain-Tallman Soda Works building to an audience that has never heard of it before. Which choice most effectively uses relevant information from the notes to accomplish this goal?

A. The Fountain- Tallman Soda Works building is a historic building in Placerville California that was built as a soda water factory but has served the community in several capacities since its construction.
B. The Fountain-Tallman Soda Works building is currently a museum with walls that are over two feet thick.
C. The Fountain and Tallman Museum started with the donation of a building to a local historical society in the late 1900s.
D. The Fountain and Tallman Museum was originally built as a soda water factory, but does not remain a factory today.

3. While researching a topic a student has taken the following notes:

  • Balsamic Vinegar of Modena (BVM) and Traditional Balsamic Vinegar (TBV) are two different products with different standards.
  • BVM has lower standards, and is much less expensive, is often aged only briefly, and contains  some ingredients that are not traditional.  It is protected under the weak European protected geographical indication (PGI).
  • TBV is aged at least 12 years, but often much longer, and has stricter controls on ingredients as well as origin as it is regulated under the strong European protected designation of origin (PDO).
  • Neither PGI nor PDO are able to be enforced in the U.S. so if consumers want to purchase real balsamic vinegar, they must be careful to read the name and look for the PDO or PGI seal on the label.

The student wants to explain to readers in the United States the variety of goods they might see labeled as “balsamic vinegar” in the grocery store. Which choice most effectively uses relevant information from the notes to accomplish this goal?

A. Both BVM and TBV can be found in the United States, but only if a discerning shopper knows where to look.
B. Because PGI and PDO are not enforced in the United States, a wide range of products, from artificially dyed vinegar to authentic TBV and BVM, may be labeled as balsamic vinegar in American stores.
C. Authentic BVM and TBV both have strict quality controls at their points of origin in northern Italy. Shoppers can be confident of authenticity by looking for PDO and PGI seals.
D. BVM and TBV are both protected in some way, with BVM having lower standards and TBV having higher standards.

4. While researching a topic a student has taken the following notes:

  • As a type of antibody, Immunoglobulin G (IgG), binds with many pathogens in the body in order to protect the body from infections brought on by viruses, bacteria, and fungi, among others.
  • IgG is the most common antibody in blood circulation and makes up around 75% of serum antibodies in humans.
  • If doctors suspect certain conditions, they may measure a patient’s levels of IgG as a diagnostic tool.
  • IgG plays a key role in newborn immunity as infants inherit IgG from their mothers through both placenta while in utero and through breast milk once born.

The student wants to include in her essay a sentence that will highlight the importance of IgG to all humans. Which choice most effectively uses relevant information from the notes to accomplish this goal?

A. IgG makes up around 75% of the average human’s serum antibodies and plays a key role in protecting the body from a wide range of infections.
B. Infants receive IgG in multiple ways from their mothers, including through the placenta and through breast milk.
C. IgG levels are used by doctors occasionally as a diagnostic indicator for certain specific conditions.  
D. While important, IgG is not the only antibody that helps support immune reaction in humans who have been infected by a virus, bacteria, or fungi.

5. While researching a topic a student has taken the following notes:

  • Mary Quant was one of the designers who helped to define 1960’s style in the United States and Great Britain.
  • Quant specialized in youthful looks with bold colors, blocky shapes, and wild patterns, though her earlier work was a bit softer and more delicate than her later work.
  • While Quant claimed to have invented the mini-skirt, some people dispute this claim, saying that the mini-skirt was invented by one of several other designers or that it was not invented by any one person, but was rather the logical end point of a continuous trend of shorter hem lengths.
  • In addition to clothing, Quant designed headwear, household goods, and personal care items, like makeup.

The student would like to explain to designers already familiar with Quant the range of her designs. Which choice most effectively uses relevant information from the notes to accomplish this goal?

A. Quant designed not just clothes we now consider classic, like the mini skirt, but also tried her hand at designing household goods, hats, and makeup.
B. Quant had a distinctive style that changed slightly over the course of her career, moving from slightly more soft and delicate to wild patterns and colors.
C. Mary Quant helped shaped style in the 1960s with her design for the now famous miniskirt.
D. With the consistently shortening hemline as a general trend, Mary Quant showed her design skill by creating the mini-skirt, a now famous look.

6. While researching a topic a student has taken the following notes:

  • While China’s imperial era is often associated with Emperors, Empress Dowager Cixi who lived from 1835 until 1908 had effective rule over the country from 1861 until her death.
  • Cixi came to power when emperor Zianfeng died, leaving Cixi’s son to inherit the throne at age five.
  • Cixi schemed to overthrow other appointed regents and served as co-regent alongside another advisor Empress Dowager Ci’an for her son’s entire life, since he was seen as a weak ruler.
  • Upon her son’s death Cixi conspired to have her young nephew placed on the throne so that she could continue in her role as regent.

The student would like to explain how Cixi managed to remain regent for so long. Which choice most effectively uses relevant information from the notes to accomplish this goal?

A. Empress Dowager Cixi reigned as regent for more than 70 years, an impressive feat for any ruler, but most especially for a female ruler of the 19th century.
B. As one of the backbones of China’s imperial era, Empress Dowager Cixi is likely remembered as the longest reigning female monarch, though she was only technically the regent.
C. In order to ensure her power, Cixi overthrew other regents so that she could control the monarchy through her son.
D. Cixi reigned for over 70 years by first serving for regent for her son and then, after his death, scheming to have her nephew put on the throne so that she could continue as regent.

7. While researching a topic a student has taken the following notes:

  • The first woman to ever be granted membership in the Entomological Society, Cynthia Evelyn Longfield was a renowned entomologist who specialized in the study of dragonflies.
  • Born in 1896, Longfield served in the Army Service Corps in World War One. After the war she traveled extensively, collecting specimens for the Natural History Museum of London.
  • In World War Two she served in the Auxiliary Fire Service.
  • Later in life she was a cataloguer at the Natural History Museum and collected 38 species of butterflies on a trip to South America. Three of those species had never been seen before.

The student wants to emphasize Longfield’s dedication to public service. Which choice most effectively uses relevant information from the notes to accomplish this goal?

A. Longfield is best remembered for her trip to South America, on which she catalogued 3 species of dragonflies which had never been seen before.
B. As the first woman to be granted membership in the Entomological Society, Longfield is well remembered in certain circles for her pioneering work with dragonflies.
C. While she is remembered for her entomological work, Longfield served her country in both World Wars I and II first in the Army Service Corps and Later in the Auxiliary Fire Service.
D. Longfield once, in her job for the Natural History Museum, collected 38 species in one single trip to South America.


8. While researching a topic a student has taken the following notes:

  • Kepler-90 is a star around 2,000 light years away from earth in the Draco constellation. It’s planetary system is quite similar to ours, as was discovered by the Kepler mission in the early 21st century.
  • The Kepler mission was designed to discover planets that orbit their stars by measuring dips in brightness of the stars as the plants cross them.
  • Kepler-90 has 8 planets just like our solar system’s sun does, however, it is thought that several of the planets that orbit Kepler-90 do not rotate on their axes, leaving them half in the dark, much like Earth’s moon.
  • Kepler-90 cannot be seen with the naked eye from Earth.

The student wants to explain the purpose of the Kepler mission and what it discovered at Kepler-90. Which choice most effectively uses relevant information from the notes to accomplish this goal?

A. The Kepler mission measured the light coming from stars including Kepler-90—a star about 2,000 light years away from earth.
B. Designed to discover planets orbiting stars, one of the Kepler mission’s findings was the eight planets orbiting Kepler-90, some of them not turning on their axes.
C. Since Kepler-90 cannot be seen with the naked eye, it took a special mission, the Kepler mission, to discover its 8 orbiting planets.
D. The Kepler mission confirmed the presence of many planets by measuring dips in light coming from far distant stars.

9. While researching a topic a student has taken the following notes:

  • Yang Xiong was a Chinese author and philosopher of the early 1st century CE. He was well-known in his time in the Han dynasty.
  • Yang gained enough praise and acclaim, that he was summoned to imperial capital where he was an officer in charge or composing fu as well as poetry for the emperor.
  • Yang did not think that the nature of humanity was inherently bad or good, but rather, he philosophized that human nature was a mixture of both.
  • His most famous work, Exemplary Sayings, is remembered for his critique of his contemporaries for their overly elaborate writings and their seeming inability to address the greater moral issues of the day.  

A student wants to give an overview of the beliefs of Yang Xiong. Which choice most effectively uses relevant information from the notes to accomplish this goal?

A. As a Chinese philosopher of the early 1st century CE, Yang Xiong wrote for the emperor on a regular basis. His work was both poetry and fu as the situation demanded.
B. Exemplary Sayings is Yang Xiongs most well remembered work because  it contained criticisms of other writers.
C. Yang held that man was duel in nature, containing a mix of good and evil and that this and other philosophical concepts should be addressed in the writing of his time.
D. Yang Xiong shunned the elaborate writing of his contemporaries.

10. While researching a topic a student has taken the following notes:

  • In many building trades like woodworking, masonry, and metalworking, workers use what is known as a combination square for a multitude of situations.
  • The combination square is made up of a ruler, and one of may different heads that can be slid over or along the ruler. A worker might use a standard head, a protractor head, or a center finding head.
  • The modern combination square dates back to the 1970s and can be used with its standard head to make 90 degree markings, miter corners, check whether a surface is level, gauge depth, and other activities vital in building.
  • With other heads, the abilities of the combination square are much more complex.

A student wants to explain to a friend a situation in which a combination square with a standard head might be useful. Which choice most effectively uses relevant information from the notes to accomplish this goal?

A. The combination square can be used for a great many things, especially if the person using it has more than one head for the tool.
B. With a standard head, a combination square could be used in building to make sure walls are level and corners meet at exact right angles.
C. Woodworkers, masons, and metalworkers all use the combination head in their daily jobs.
D. Depending on the situation, a builder may switch out the heads of a combination square to complete different tasks.

Solutions

1. D. The question requires that the answer explain why the entire temple can’t be seen today. While the notes given do include information on an early fire, it also explains that the temple was restored after that first fire in 64 CE. That would point us to the second fire in 363 CE as the final destruction of the temple and the reason why it is not visible today, making answer option D the correct answer. Answer option A explains why parts are visible today, but not why the rest of the temple is not visible. Answer option B explains the earlier destruction prior to the restoration. Answer option C includes information that does not answer the question.

2. A. The question asks for a general introduction to the building for an audience that is completely unfamiliar with the building. This makes answer A the best option as it gives the best broad background for the building. Option B gives just one detail of the building, not an introduction. Option C gives an introduction to the museum, not the building. Option D gives the origin of the building, but no information as to the rest of its history or relevance today.

3. B. The question asks about the variety found on the stores in the U.S. The final note of the set indicates that, due to lack of protections, in addition to real BVM or TBV there are many knock off products in the U.S. This makes option B the best to answer the question. Option A does not explain the variety available beyond BVM and TBV. Option C explains how to find BVM and TBV, but not what else is on shelves. Option D explains the protections on BVM and TBV but not the variety of other goods available.

4. A. The question asks about the importance of IgG to all humans. Option A best explains that IgG plays a key role in protecting all humans from illnesses. Option B explains how infants get IgG, but not its role. Option C explains how it can be a useful diagnostic, but only for people with certain conditions, not for all humans. Option D implies that IgG is plays only a part in immunity and therefore does not highlight its importance to all humans.

5. A. The question asks for an answer that assumes the reader already knows something about Mary Quant and that explains her range beyond just fashion design. Answer A fulfills this prompt by explaining that she designed household goods, hats, and makeup. Option B is incorrect as it explains a slight and gradual change over time, not a wide range of designs. Option C is incorrect as it gives a general introduction to Quant, but does not explain the range of her designs. Option D is incorrect as it addresses just one of her designs instead of showing her range.

6. D. The question asks for an explanation of how Cixi managed to rule for so long as regent. Option D explains this the best by telling the reader that Cixi ruled not just during her son’s childhood, but also by placing another child on the throne after her son’s death. Options A and B are incorrect as they do not explain how she ruled for 70 years. Option C is incorrect as it explains how Cixi came to power, but not how she retained that power for 70 years.

7. C. The question asks for information on Longfield’s public service, not her entomological activities. This makes C the best option and A, B, and D incorrect.

8. B. The question asks for the purpose of the Kepler mission and what was discovered at Kepler-90. Option B is the best answer as it explains that Kepler was designed to discover planets orbiting stars (its purpose) and what it found at Kepler-90 (8 orbiting plants, some of which do not turn on their axes). Option A explains what Kepler did, but not what it discovered at Kepler-90. Option C explains why the Kepler mission was necessary, but not what its purpose was. Option D does not explain specifically what was discovered at Kepler-90.

9. C. The question asks about Yang Xiong’s beliefs. This is best summed up in option C. Option A explains his work. Option B explains why his work Exemplary Sayings is remembered. Option D explains his attitude toward his contemporaries.

10. B. The question asks for a situation in which a combination square with a standard head would be used. We learn in the notes that with a standard head, the combination square can be used to mark 90 degree (right) angles, miter corners, check for level surfaces, and gauge depth. This makes answer B the best option and the other answer incorrect.

Digital SAT Poetry Practice Reading Questions

1. A student claims that Will Carleton’s Poem “Autumn Days” contrasts the sweetness of some autumn days in the first stanza with a far different type of autumn days in the second stanza. What pair of lines from the first and second stanzas respectively best illustrate this claim?


A. O’er the dreamy, listless haze/O’er the cheerless, withered plain.
B. Yellow, mellow, ripened days/ Shivering, quivering, tearful days.
C. And the sombre, furrowed fallow/ Woefully and hoarsely calling.
D. Winking at the blushing trees/On thy scanty vestments falling.

2. The following is an excerpt from the poem “We Wait” by Will M. Carleton

Or if upon the field of war we stand,
And sword with sword for mastery we mate,
Grim Death, and radiant Glory, hand in hand,
Approaching us with silent step we see;
And one of them, we vow, for us must be;
Bravely we strive to win renown’s estate,
And still we wait.

And when we grope within the gloom of age,
When our few steps grow feeble and sedate,
We cast our eyes back o’er a blotted page;
We peer among the pictures of the past,
We gaze upon the future, overcast;
Our musings all with hopes and fears we freight;
And still we wait.

Which choice best states the main purpose of the text?


A. To illustrate the abeyancy of life, even as death approaches.
B. To force the reader to consider his own fate.
C. To illustrate the futility of war.
D. To explain the purposelessness of life.

3. The following is an excerpt from the poem “We Hope” by Will M. Carleton


Then we yearn and call for comfort; but no comfort comes unto us,
And we wrap ourselves in sadness, and Despair goes thrilling thou’ us;
And the darkness gathers round us, with its horrors, half-unspoken,
And we pray again for succor: that the fearful spell be broken,
With the light of something shining, be it only but a ray.

Then within our hearts a blossom, from the dreary mould is springing,
Then the birds of Hope make music, with their sweet and cheerful singing;
Then, upon the great clouds gazing, we discern their silver lining,
And at last, through veils of blackness, bursts the sunbeam’s glorious shining,
And upon our raptured vision beams the light of perfect day

Which choice best describes the function of the underlined portion in the text as a whole?


A. It minimizes the role of hope to “but a ray”.
B. It firmly emphasizes the despair of the writer.
C. It clarifies the despair that was described earlier in the passage.
D. It introduces a visual for hope that will be further built upon in the poem.

4. The following is an excerpt from the poem “The House Where We Were Wed” by Will M. Carleton

I’ve been to the old farm-house, good-wife,
Where you and I were wed;
Where the love was born to our two hearts
That now lies cold and dead.
Where a long-kept secret to you I told,
In the yellow beams of the moon,
And we forged our vows out of love’s own gold,
To be broken so soon, so soon!

Which choice best states the main purpose of the text?


A. To tell someone of a trip made, in the light of a broken relationship.
B. To set the stage for a future argument.
C. To argue that marriage is a fruitless endeavor.
D. To help the reader feel the author’s pain after the death of his wife.

5. The following is an excerpt from the poem “Apple Blossoms” by Will M. Carleton


Naught within her eyes he read
That would tell her mind unto him;
Though their light, he after said,
Quivered swiftly through and through him;
Till at last his heart burst free
From the prayer with which ‘twas laden,

And he said, “When wilt thou be
Mine for evermore, fair maiden?”


Which choice best describes the function of the underlined portion in the text as a whole?


A. To clarify the emotional source of the following quotation.
B. To explain a medical condition from which he is suffering.
C. To show the religious fervor with which he lives his life.
D. To build on the previous description of her eyes.

6. An instructor claims that “Lines Written in Early Spring” contains the introspective thoughts of the author. Which quotation from the poem best supports this claim?


A. “And ‘tis my faith that every flower/Enjoys the air it breathes.”
B. “The birds around me hopp’d and play’d/ Their thoughts I cannot measure”
C. “In that sweet mood when pleasant thoughts/ Bring sad thoughts to the mind.”
D. “I heard a thousand blended notes/While in a grove I sat reclined.”

7. The following is an excerpt from “The Dungeon” as published in Lyrical Ballads With a Few Other Poems.


And this place our forefathers made for man!
This is the process of our love and wisdom,
To each poor brother who offends against us—
Most innocent, perhaps—and what if guilty?
Is this the only cure? Merciful God?
Each pore and natural outlet shrivell’d up
By ignorance and parching poverty,
His energies roll back upon his heart,
And stagnate and corrupt; till changed to poison,
They break out on him, like a loathsome plague-spot;
Then we call in our pamper’d mountebanks—
And this is their best cure! uncomforted
And friendless solitude, groaning and tears,
And savage faces, at the clanking hour,
Seen through the steams and vapour of his dungeon,
By the lamp’s dismal twilight! So he lies
Circled with evil, till his very soul
Unmoulds its essence, hopelessly deformed
By sights of ever more deformity!


Which choice best states the main purpose of the text?


A. It examines the purpose of a dungeon form the point of view of a jailor.
B. It critiques a solution that society has found to a common issue.
C. It asks a question about the worth of humanity.
D. It sheds a negative light on how humanity handles a problem.

8. The following is an excerpt from the poem “Expostulation and Reply”. The author speaks to his friend, Matthew:

“The eye it cannot chuse but see,
“We cannot bid the ear be still;
“Our bodies feel, where’er they be,
“Against, or with our will.

“Nor less I deem that there are powers,
“Which of themselves our minds impress,
“That we can feed this mind of ours,
“In a wise passiveness.

“Think you, mid all this mighty sum
“Of things for ever speaking,
“That nothing of itself will come,
“But we must still be seeking?

“—Then ask not wherefore, here, alone,
“Conversing as I may,
“I sit upon this old grey stone,
“And dream my time away.”


Which choice best describes the function of the underlined portion in the text as a whole?


A. It questions the author’s purpose.
B. It asks Matthew a philosophical question.
C. It highlights a subject for which the author is passionate.
D. It explains an earlier statement.

9. A student reads “Old Man Travelling; Animal Tranquility and Decay, A Sketch” and observes that the old man in the poem seems at great peace with his life. Which of the following excerpts from the poem best supports this claim?

A.”Sir! I am going many miles to take/A last leave of my son, a mariner,/ Who from a sea-fight has been brought to Falmouth/ And there is dying in an hospital.”
B. “He travels on, and in his face, his step,/ His gait, is one expression;/ every limb,/ His look and bending figure, all bespeak/ A man who does not move with pain.”
C. “He is one by whom/ All effort seems forgotten, one to whom/ Long patience has such mild composure given/ That patience now doth seem a thin, of which/He hath no need. He is by nature led.”
D. “The young behold/ With envy, what the old man hardly feels./ I asked him whither he was bound, and what/ The object of his journey.”

10. The following is the poem “Why Do Ye Call The Poet Lonely?” By Archibald Lampman

Why do ye call the poet lonely,
Because he dreams in lonely places?
He is not desolate, but only
Sees, where ye cannot, hidden faces.


Which choice best states the main purpose of the text?

A. It asks and answers a question about those who write poetry.
B. It hypothesizes as to what makes people want to write poetry.
C. It gives an explanation as to why much poetry is sad.
D. It opens up the reader to ask questions of poets.

Answer Explanations

  1. B. The question asks for evidence to show a contrast between two different types of autumn days. Answer option B shows two types of days in the fall, one that is yellow (leaf color), mellow (meaning calm), and ripened (as the harvest on the vine). The other is shivering and quivering (cold) as well as tearful (raining). This makes option (B) the best answer. (A) and (C) are incorrect as they only describe one setting, not two. Option (D) is incorrect as it does not describe a day at all, but rather leaves falling.
  2. A. In this poem we see at the end of both stanzas “And still we wait”. The author describes this waiting even when death is near and in every situation. He seems to be telling the reader that life is just a game of waiting. This purpose is best described in answer option (A) since the word abeyancy describes a situation of disuse, suspension, or waiting. Answer option (B) is incorrect for, while a reader may consider his own fate, this does not seem to be the main purpose of the poem. Answer (C) is incorrect as the author seems to be describing the futility of all things, not just war. Answer (D) is incorrect as the poem gives no explanation as to the purposelessness of life that the author sees.
  3. D. In the underlined section the author introduces the idea of visual light. In the second stanza he builds on this image of a single ray of light by describing the emerging “sunbeam’s glorious shining” and “the light of perfect day.” In this way, the light represents a visual image of the hope bursting through dark clouds of sadness. This makes option (D) the best answer. The other answer options do not appropriately understand that the visual light represents hope and that it grows to a sunburst in stanza 2.
  4. A. The opening lines of this poem tells the author’s “good-wife” that he has “been to the old farm house… where you and I were wed”. He is telling her he has gone back to where their marriage started. The end of this first stanza sheds light on the current status of their relationship “we forged our vows… to be broken soon.” Their marriage vows, and thus their relationship, are broken. This makes option (A) the best answer. We have no evidence of a future argument, so option (B) is incorrect. The author knows that his marriage is over, but does not try to say that all marriages are pointless, making option (C) incorrect. There is no evidence that the wife is dead, just that they are separated, making option (D) incorrect.
  5. A. The underlined portion describes his heart “bursting free” from “the prayer with which ‘twas laden” thereafter the man spills his heart to the girl and asks her to stay with him forever. These words of his are from his heart. This makes (A) the best option. The underlined portion explains the source of his ardent outburst. Option (B) is incorrect as his heart is not literally bursting, but figuratively bursting. Option (C) is incorrect as there is no religious undertones to the underlined section. Option (D) is incorrect as the underlined portion describes the source of his words, not her eyes.
  6. C. To be introspective is to think about one’s self and one’s own thoughts and emotions. Answer option (C) is the only answer that gives us a glimpse into the author’s mind to support the teacher’s claim that the author is “introspective”. Answer (A) describes the author’s belief about flowers. Answer (B) and (D) simply describe events that happen.
  7. D. This poem describes what happens to men thrown into a dungeon. The author describes the mental effects of being imprisoned in a very negative way saying things like that the prisoner’s soul is “hopelessly deformed” and that his energy turns inward “till changed to poison”. This makes option (D) the best answer. The author is shedding negative light on how the world deals with the problem of crime. Answer option (A) is incorrect as the poem is not from the point of view of a jailor. Answer (B) is incorrect as a dungeon is not a solution to but rather a punishment for crime. Answer (C) is incorrect as the author does not question the worth of humanity in general, but rather the worth of the dungeon itself.
  8. B. The underlined section, when paraphrased in modern English, would be something like “do you ever think in the middle of all this craziness that we just have to keep going and going even though we’re not going to get anywhere?” This is equivalent to asking if life has any meaning, a deeply philosophical question. This makes answer (B) the best option. This questions the purpose of “seeking”, not of the author, so answer option (A) is incorrect. Answer option (C) is incorrect as we have no evidence that the author is particularly passionate about this topic. Answer option (D) is incorrect as the underlined passage may build on a previous statement, but it does not explain it.
  9. C. The question is asking for evidence that the man feels great peace. Answer option (A) describes a situation in which it would make sense to have little or no peace. Answer option (B) describes the man as having no pain, which is not the same as being at great peace. Answer option (D) says that the young envy him, but that does not necessarily mean they envy any great peace he might have. This leaves option (C) in which the man is described as having forgotten any and all effort, who has so mild a personality that he needs no patience, and who is led by nature. This gives the best evidence that the man is at peace.
  10. A. The first two lines of this short poem ask why the reader calls poets lonely. The second two lines responds to the question, explaining that poets are not lonely, but rather see hidden faces where we cannot. This makes answer option (A) the best solution.

Digital SAT Text Structure and Purpose Reading Practice Questions

The text from these questions is adapted from the book “How to Know the Wildflowers,” by Frances Theodora Parsons.

1. Pliny tells us that the anemone of the classics was so entitled because it opened at the wind’s bidding. The Greek tradition claims that it sprang from the passionate tears shed by Venus over the body of the slain Adonis. At one time it was believed that the wind which had passed over a field of anemones was poisoned and that disease followed in its wake. Perhaps because of this superstition, the flower was adopted as the emblem of sickness by the Persians. Surely our delicate blossom is far removed from any suggestion of disease or unwholesomeness, seeming instead to hold the very essence of spring and purity in its quivering cup.

What choice best states the function of the underlined sentence in the text as a whole?
A. It explains how a plant caused illness historically.
B. It clarifies what Pliny believes about the use of the plant.
C. It presents a hypothesis as to the origin of a historic symbol.
D. It argues against a commonly held belief.

2. Ginseng is well known by name but is yearly becoming more scarce. The aromatic root is so greatly valued in China for its supposed power of combating fatigue and old age that it can only be gathered by order of the emperor. The forked specimens are believed to be the most powerful, and their fancied likeness to the human form has obtained for the plant the Chinese title of Jinchen (from which ginseng is a corruption), and the Indian one of Garntoguen, both of which, strangely enough, are said to signify like a man.


What choice best describes the function of the underlined information in text as a whole?
A. It explains the origin of an English word.
B. It clarifies a common mispronunciation of a Chinese word.
C. It gives the reader irrelevant information.
D. It allows the reader to visualize the plant.

3. He who seeks the cool shade of the evergreens on a hot July day is likely to discover the nodding wax-like flowers of this little plant. They are delicate and pretty, with a background of shining leaves. These leaves when young have a pleasant aromatic flavor similar to that of the sweet birch; they are sometimes used as a substitute for tea. The bright red berries are also edible and savory and are much appreciated by the hungry birds and deer during the winter. If not thus consumed, they remain upon the plant until the following spring when they either drop or rot upon the stem, thus allowing the seeds to escape.


What choice best describes the function of the underlined sentence in the text as a whole?
A. It concludes the text with a detail about the safety of berry consumption.
B. It helps the reader understand how evergreens spread.
C. It clarifies the nutritional value of berries to the birds and deer.
D. It builds on the claim of the previous sentence.

4. The common knotweed, P. aviculare, which grows in such abundance in country dooryards and waste places, has slender, often prostrate, stems, and small greenish flowers, which are clustered in the axils of the leaves or spike at the termination of the stems. This is perhaps the “hindering knotgrass” to which Shakespeare refers in the “Midsummer Night’s Dream,” so terming it, not on account of its knotted trailing stems, but because of the belief that it would hinder the growth of a child.

What choice best describes the function of the underlined selection in the text as a whole?
A. It introduces a term from literature that will later be clarified.
B. It provides direct evidence for a previous claim.
C. It gives the exact words of a noted botanist.
D. It explains the use of a previously explained.

5. Despite the difference in the spelling of the names, it has been suggested [about the pokeweed] that the plant was called after President Polk. This is most improbable, as it was common throughout the country long before his birth, and its twigs are said to have been plucked and worn by his followers during his campaign for the Presidency.

What choice best describes the function of the underlined sentence in the text as a whole?
A. It provides evidence as to the origin of the name Pokeweed.
B. It explains where a term originated.
C. It presents chronological information to assuage the reader’s doubts.
D. It refutes a claim as to the origin of a term.

6. This [Queen Anne’s Lace] is believed to be the stock from which the garden carrot was raised. The vegetable was well known to the ancients, and we learn from Pliny that the finest specimens were brought to Rome from Candia. When it was first introduced into Great Britain is not known, although the supposition is that it was brought over by the Dutch during the reign of Elizabeth. In the writings of Parkinson, we read that the ladies wore carrot leaves in their hair in place of feathers. One can picture the dejected appearance of a ball-room belle at the close of entertainment.


What choice best describes the function of the underlined sentence in the text as a whole?
A. It presents literary evidence of a novel use for carrots.
B. It explains the depth and breadth of a tradition.
C. It helps the reader visualize Queen Elizabeth.
D. It gives evidence for the previous thought.

7. Some October day, as we pick our way through the salt marches which lie back of the beach, we may spy in the distance a thicket which looks as though composed of such white-flowered shrubs as belongs to June. Hastening to the spot we discover that the silky-tufted seeds of the female groundsel tree are responsible for our surprise. The shrub is much more noticeable and effective at this season than when—a few weeks previous—it was covered with its small white or yellowish flower-heads.

What choice best describes the function of the underlined sentence in the text as a whole?
A. It sets the scene for a shocking discovery later in the text.
B. It describes an interesting discovery that is later explained in the text.
C. It sets the scene for an unusual activity.
D. It helps the reader visualize a normal setting.

8. The yellow lady’s slipper usually blossoms in May or June, a few days later than its pink sister, C. acaule. Regarding its favorite haunts, Mr. Baldwin says: “Its preference is for maples, beeches, and particularly butternuts, and for sloping of hilly ground, and I always look with glad suspicion at a knoll covered with ferns, cohoshes, and trilliums, expecting to see a clump of this plant among them. Its sentinel-like habit of choosing ‘slightly places’ leads it to venture well up on mountain sides.”


What choice best describes the function of the underlined phrase in the text as a whole?
A. It explains why the author does not believe Mr. Baldwin.
B. It gives a reason why Mr. Baldwin is suspicious of the yellow lady’s slipper.
C. It explains where Mr. Baldwin expects to find yellow lady’s slipper.
D. It explains what plants Mr. Baldwin expects to find on knolls.

9. If Emerson’s definition of a weed, as a plant whose virtues have not yet been discovered, be correct, we can hardly place the dandelion in that category, for its young sprouts have been valued as a pot-herb, its fresh leaves enjoyed as a salad, and its dried roots used as a substitute for coffee in various countries and ages. The feathery-tufted seeds which form the downy balls beloved as “clocks” by country children, are delicately and beautifully adapted to dissemination by the wind. The common name is a corruption of the French dent de lion.


What choice best describes the function of the quotation marks around the underlined word in the overall structure of the text?
A. They place emphases on the word.
B. They show that the word is a direct quote from Emerson.
C. They show that, while they are treated like clocks, dandelions are not actual clocks.
D. They show that the word inside is not truly part of the sentence.

10. “Too well known as a pernicious weed which is difficult to extirpate,” is the scornful notice which the botany gives to Common St. John’s-wort whose bright yellow flowers are noticeable in waste fields and along roadsides nearly all summer. Its rank, rapid growth proves very exhausting to the soil, and every New England farmer wishes it had remained where it rightfully belongs—on the other side of the water.


What choice best describes the function of the underlined phrase in the overall structure of the text?
A. It explains the origin of St. John’s-wort.
B. It explains the origin of the farmers.
C. It clarifies the location of the St. John’s-wort.
D. It clarifies the location of the farmers.

Answer Explanations

1. C. In the passage, we learn that a story of wind passing over a field of anemones causing the wind to be poisoned and lead to disease. The next sentence uses the word “perhaps” to speculate that this may have been where an ancient symbol of disease originated. This makes (C) the best answer option. It is not (A) since this is a story, not a historical fact. It is not (B) as this is not something Pliny believed, but rather something modern historians hypothesize. It is not (D) since it does not argue against this belief.

2. A. We learn in the first sentence of the text that the text is discussing the plant Ginseng. The parenthetical information (the information between the parenthesis) clarifies that from this name comes “a corruption” of the Chinese title of the plant: Jinchen. This makes option (A) the best answer. Answer (B) is incorrect since Ginseng is an English word with a Chinese origin, not a mispronunciation of a Chinese word. Answer (C) is incorrect since the information is not irrelevant to the discussion. Answer (D) is incorrect as it does not give any idea of Ginseng’s visual appearance.

3. B. In the text, we learn that evergreens have bright berries that are often eaten by birds or deer. The underlined sentence then concludes that if they are not eaten, they fall or rot, thus spreading the seeds. With the spread of seeds, the evergreens also spread. This makes option (B) the best answer. Option (A) is incorrect as the underlined portion does not give any details about the safety of consuming berries. Option (C) is incorrect as it does not explain the nutritional value of eating the berries. Option (D) is incorrect as the previous sentence does not make a claim about uneaten berries.

4. A. The term “hindering knotgrass” is an unknown term when first introduced. The rest of the sentence explains that this was a term used by Shakespeare and that the author believes that Shakespeare may have been referring to the plant knotweed and why. This makes answer (A) the best option. (B) is incorrect as “hindering knotgrass” is not evidence of anything. (C) is incorrect as Shakespeare is not a noted botanist. (D) is incorrect as the term is not previously explained term.

5. D. The second sentence explains why it isn’t possible that pokeweed was named after President Polk. This makes sentence (D) the best option—the term “pokeweed” didn’t come from President Polk. (A) and (B) are the exact opposite of what the sentence is doing. (C) is incorrect as it does not assuage doubts.

6. D. In the previous sentence the author states that it is thought that the carrot came to England during the reign of Elizabeth. The next sentence gives evidence for this by explaining that ladies were wearing carrot leaves in their hair, meaning the carrots must have been brought over. Option (A) is incorrect as it is historical evidence, not literary evidence. Option (B) is incorrect as there is no evidence that the wearing of carrot leaves is a tradition, rather it seems to have been a fad. Option (C) is incorrect as the sentence does not describe how Queen Elizabeth looked, but rather an accessory that she might have worn.

7. B. The sentence in question notes that “we” are out walking in October, yet see plants in the distance that look like they have June flowers on them. This is an interesting discovery. The rest of the text goes on to explain the plant that has these curious late-season flowers. This makes (B) the best option. (A) is incorrect as we can’t describe this as a shocking discovery. Option (C) is incorrect as the plant is unusual, not the activity. (D) is incorrect as the sentence helps the reader visualize an unusual setting (with flowers in October) not a normal setting.

8. C. The underlined sentence explains that the speaker, Mr. Balwin, always looks with “glad suspicion” at certain areas, “expecting to see a clump of this plant” (C. Acaule). This means he expects to find C. Acaule among the other plants mentioned, making option (C) the best answer. Answer (A) is incorrect as these are Mr. Baldwin’s words, not suspicion on the part of the author. Option (B) is incorrect as Mr. Baldwin is not suspicious of a plant, but rather looks “suspiciously” at where the plant grows. Option (D) is incorrect as he doesn’t expect knolls in general to have certain plants, but when they have fens, cohoshes, and trilliums, he expects to find yellow lady’s slipper as well.

9. C. In this instance the quotation marks are being used as a person might use what we call “air quotes” to show that the speaker does not mean the literal use of the word. (C) is the best option as the author does not mean to say that the dandelions are being used as literal clocks. Option (A) is incorrect as the quotes are used to show non-standard use of the word, not to place emphasis. Option (B) is incorrect as the word is not a direct quote. Option (C) is incorrect as the word “clocks” is indeed part of the sentence.

10. A. The New England farmer wishes that the plant had remained “on the other side of the water”. In this case, there is only one body of water that all New England farmers could refer to in such a vague way—the Atlantic Ocean. Therefore, St. Joh’s Wort must have origins somewhere on the other side of the ocean. This makes option (A) the best answer. Option (B) is incorrect as it is the origin of the plant, not the farmers, to which is being referred. Option (C) is incorrect as St. John’s wort has spread, and so its original location is no longer its only location. It is not option (D) as the farmers are in New England.

Digital SAT Standard English Conventions Practice Questions

1. Despite a fearsome cough that for two months racked his ___________ three ribs in a particularly violent episode — the decrepit nobleman had greedily held fast his grip on life.

Which choice completes the text so that it conforms to the conventions of Standard English?
a. torso — cracking
b. torso, cracking
c. torso: cracking
d. torso crackin

2. Naturally, Sweden’s aggressive new policies came with a price tag — particularly its massive investment in ___________ creating a large federal deficit, in 1934 Sweden became the first country to fully emerge from the Depression and foreign creditors were quickly recompensed.

Which choice completes the text so that it conforms to the conventions of Standard English?
a. public work’s projects. But despite
b. public works projects: but despite
c. public works projects. But despite
d. public work’s projects; but despite

3. Although Mendeleev’s table eliminated the chemical inconsistencies of the telluric helix, ___________
by standard chemical terminology, large pockets of the scientific community remained resistant to the notion of periodicity.

Which choice completes the text so that it conforms to the conventions of Standard English?
a. and were accompanied
b. and was accompanied
c. and accompanied
d. and accompanies

4. The very scope of the action dictates the terms of this contract and renders the least modification of them inadmissible, something making them null and void. Thus, although perhaps they have never been stated ___________ the same everywhere and tacitly conceded and recognized everywhere.


Which choice completes the text so that it conforms to the conventions of Standard English?
a. in so many words, they are
b. in so many words: they are
c. in so many words they are
d. in, so many words, they are

5. For the next forty years, Michelangelo continued to introduce the stark, complex, and disturbing motifs found in his Library to the architecture, sculptures, and paintings ___________ the Mannerist movement.

Which choice completes the text so that it conforms to the conventions of Standard English?

a. that would later come, to christen
b. that would later come to christen
c. that would later come too christen
d. that would later come for christen

6. Remarkable, ___________ through the frosted window to where Lady Cress was dancing a quadrille. She’s gone along with this the whole while.

Which choice completes the text so that it conforms to the conventions of Standard English?

a. thought Forsythe: gazing back
b. thought Forsythe gazing back
c. thought Forsythe gazing, back
d. thought Forsythe, gazing back

7. Apart from extending unemployment benefits, the government also reinforced its agricultural industry by subsidizing farms when necessary ___________ to protect the price of domestically grown crops.

Which choice completes the text so that it conforms to the conventions of Standard English?
a. and established tariffs
b. and having established tariffs
c. and established tariff
d. and establishing tariffs

8. Unfortunately, because Chancourtois inexplicably included several polyatomic ions on the helix, and published his report using ___________ was largely ignored.

Which choice completes the text so that it conforms to the conventions of Standard English?
a. geological, rather than chemical terminology, his discovery
b. geological rather than chemical terminology his discovery
c. geological rather than chemical terminology, his discovery
d. geological, rather than chemical terminology his discovery

9. The social contract’s terms, when they are well understood, can be reduced to a ___________ member alienates himself totally to the whole community together with all his rights.

Which choice completes the text so that it conforms to the conventions of Standard English?
a. single stipulation: the individual
b. single stipulation, the individual
c. single stipulation the individual
d. single stipulation: the individual,

10. Disquieting in design, and utterly without regard for classical convention, the vestibule is an exceedingly high, narrow room, whose massive dark and imposing staircase seems to push visitors outward, ___________

Which choice completes the text so that it conforms to the conventions of Standard English?
a. rather than invited them inside.
b. rather than offering an invitation to come in.
c. rather than giving to guests the opportunity to enter.
d. rather than inviting them inside.

11. Many of the pieces in the room are composed of natural, local materials of Latin America. Other pieces throughout the exhibition ___________ traditional design but are reconstructed out of contemporary materials (like aluminum or fiberglass).

Which choice completes the text so that it conforms to the conventions of Standard English?
a. reflect
b. will reflect
c. reflects
d. has reflected

12. Cystic Fibrosis is an inherited lung disease affecting the secretory ___________ responsible for producing mucus and sweat. This disease causes debilitating breathing and digestive deficiency.

Which choice completes the text so that it conforms to the conventions of Standard English?
a. glands, they are
b. glands; which are
c. glands, which is
d. glands, which are

13. Plato argues that taboos and societal norms influence our ability to change the form of societal ___________
that conventional ideas and historical traditions can prevent civilizations from being able to imagine new systems of organization.

Which choice completes the text so that it conforms to the conventions of Standard English?
a. structure, insofar
b. structure insofar
c. structure; insofar
d. structure. Insofar

14. Fourteen years have come and gone, bringing with them 256 wins, 9 All-Star games, and 2 World Series of my very own. My elbow is shot, my shoulder is in tatters, and my back barks at me when I get out of bed every morning, ___________ I can walk away from this game with my head held high and my heart full of pride.

Which choice completes the text so that it conforms to the conventions of Standard English?
a. for
b. but
c. and
d. so

15. The Cannibalism Manifesto, written by Oswald de Andrade and published in 1928, critiqued European colonialism and expressed that Brazil’s greatest strength ___________ to “cannibalize” other cultures by actively selecting and reappropriating the cultural principles of others in a new way.

Which choice completes the text so that it conforms to the conventions of Standard English?
a. was their ability
b. is their ability
c. is its ability
d. was its ability

16. Moderno explores the progression of interior design and its relation to daily ___________ also questioning how design reflected the political climate of the time.

Which choice completes the text so that it conforms to the conventions of Standard English?
a. life, but
b. life: but
c. life, while
d. life; while

17. As sweat is produced from sweat glands, cells near the skin absorb the sodium, chloride, and other ions produced ___________ cystic fibrosis transmembrane conductance regulator (CFTR) does not allow for absorption to occur. This, in turn, can lead to grave complications.

Which choice completes the text so that it conforms to the conventions of Standard English?
a. by the glands. On the other hand, a missing
b. by the glands but, a missing
c. by the glands, however, a missing
d. by the glands. However, a missing

18. Plato seeks to replace the kinship system with a new model in which people who aren’t blood relatives still interact in ways now reserved for familiar ___________ forms a communal family: “every time he meets any of them, he will assume he is meeting his brother, or sister, or, mother, or son, or daughter — or the child or parent of one of these.”

Which choice completes the text so that it conforms to the conventions of Standard English?
a. relations, in his vision, the city-state
b. relations. In his vision, the city-state
c. relations; in his vision the city-state
d. relations in his vision. The city-state

19. Life, we’ve seemingly ___________ a commodity most effectively measured in years.

Which choice completes the text so that it conforms to the conventions of Standard English?
a. arbitrarily decided, is
b. arbitrarily decided: is
c. arbitrarily decided is
d. arbitrarily, decided is

20. Money that was previously pumped into developing markets is now being funneled toward a much ___________ the general feeling among the electorate is that the country is as healthy as ever.

Which choice completes the text so that it conforms to the conventions of Standard English?
a. safer, yet no less promising, Stockholm, and
b. safer yet no less promising Stockholm, and
c. safer, yet no less promising Stockholm, and
d. safer yet no less promising Stockholm and

Answer Explanations

  1. A. When there is extra information interrupting an independent clause two commas or two dashes may be used to show the reader that the information is extra. In this sentence, the phrase “cracking three ribs in a particularly violent episode” is extra. Since there is a dash at the end of that extra information there must be a matching dash at the beginning, making (A) correct. Answer option (B) would be correct if there were a comma after the word episode. Answer options (C) and (D) both are incorrect ways of showing that there is extra information in a sentence.
  2. C. Answers (A) and (D) can be eliminated since they use the possessive “works” instead of the plural “works”. The works don’t own anything, so “work’s” is incorrect. A colon is used to connect an independent clause to a clarification or explanation, so (B) is incorrect. Since the punctuation is connecting two complete sentences, or independent clauses, a period is the best option.
  3. B. This question is, in part, asking us to select the verb that would best fit in the blank. On questions like this it is important to identify the subject of the verb. What is the “thing” that is “doing the action”. In this case, what is being accompanied or accompanying the standard chemical terminology. The “table” is the subject, but the table isn’t accompanying anything, rather, it is being accompanied by the standard chemical terminology. Since we are in the past tense (as shown by the word “eliminated”) “and was accompanied” (B) is our best option since it is singular to match “table” and doesn’t force the table to do the accompanying.
  4. A. Within the sentence the phrase “although perhaps they have never been stated in so many words” would be considered extra information, since it is interrupting “thus they are the same everywhere and tacitly conceded and recognized everywhere.” Extra information is shown by putting a comma or dash before (after the word “thus”) and after (after the words “words”) it. Answer option (A) is the right answer. Answer options (B) and (C) do not put a comma after the word “words” and answer option (D) has a non-necessary extra comma after the word “in”.
  5. B. Since there are no clauses being connected and no extra information beginning or ending in the blank, no punctuation is needed. This eliminates option (A). Option (C) is incorrect as it uses “too” which is synonymous with “also”. Answer option D is incorrect since “for” is the incorrect word choice. Option (B) is correct since it lacks any punctuation and has the correct “to”.
  6. D. Within the sentence, the phrase “thought Forsythe” should be separated from the rest of the sentence since it is describing where the quotation “remarkable” is coming from. In English this is generally done by putting a comma before and after the descriptor. In this case, we have a comma after “remarkable”, so we need a matching comma after “Forsythe”. This makes option (D) correct and the other options incorrect. Another way to figure this out would be to read the sentence “out loud” in your head and hear the pause after “Forsythe”.
  7. D. The government “reinforce its agricultural industry” by doing two things. The first thing they did was “subsidizing farms” the second thing they do, therefore, should have the same verb tense. This makes (D) “and establishing tariffs” correct and the other answers incorrect.
  8. C. In order to figure out where commas should go, start by identifying clauses and looking for extra information. There is no extra information in this sentence, but there are two separate clauses. The second clause “his discovery was largely ignored” is independent, since it can be a sentence on its own. The first clause is dependent. Independent and dependent clauses are connected with a single comma. This makes answer option ( C) correct as it has a single comma after the end of the first clause. (A) uses two commas around a phrase that is not extra, (B) is a run on sentence since the two clauses are not correctly connected, and (D) puts the comma in the middle of the dependent clause instead of between the independent and dependent clauses.
  9. A. A colon connects an independent clause to an explanation or clarification of that independent clause. In this case, the second part of the sentence is clarifying what the “single stipulation” is from the first part of the sentence. This means that a colon is the most appropriate punctuation to have after the word “stipulation”. Thus, options (B) and (C) are incorrect. Option (D) is incorrect as there is no reason to put a comma after the word “individual”. There are no clauses being connected or extra information in this place.
  10. D. Good writing gets the point across as clearly and directly as possible. Answers (B), and (C) say the same thing in less succinct ways and are therefore incorrect. (A) uses the incorrect form of the verb “to invite”.
  11. A This passage is in the present as can be seen in the use of the very “are” twice. Therefore, the verb will be in the present. (B) is in the future and (D) is in the past, making both options incorrect. (C) uses the singular “reflects” to refer to multiple pieces. The plural tense “reflect” (A) is the correct option. If the difference between singular and plural verb tense confuses you, just listen: would you say “the pieces reflect” or the “the pieces reflects”?
  12. D. Remember to identify cluses as you make decisions about punctuation. A comma can connect a dependent and independent clause; a semicolon connects two independent clause. (A) is incorrect as it is connecting two independent clauses with just a coma. (B) is incorrect as it connects an independent and dependent clause with a semicolon. (C) is incorrect as it uses the singular “is” to refer to the plural “glands”. (D) is the best option as it uses a comma to connect an independent and dependent clause and uses the plural “are” to refer to the plural “glands.”
  13. B. Since there is only one clause, no punctuation is needed. (A) is incorrect as a comma would connect a dependent and independent clause. (C) and (D) are incorrect as they would connect two independent clauses.
  14. B. Think about the author’s intended meaning. This is a situation where the author is setting up a contrast between the negatives of the physical toll on his body with the positives of the pride he feels. In order to show this contrast “but” is the best option. The other answer options to not make logical sense with the contrast set up in the passage.
  15. C. The subject of the very in question is the strength of Brazil, a singular subject. Consequently, answer options (A) and (B) are incorrect as they us the plural “their” to refer to the strength. (D) is incorrect as it is in the past tense.
  16. C. Since the second clause is not independent, options (A) and (D) are incorrect. Both a comma with one of the FANBOYS and a semi-colon connect two independent clauses. Answer option (B) is incorrect since a colon is designed to go after an independent clause but before a clarification or explanation. The second part Is not clarifying the first. This leaves option (C) where a comma connects an independent and dependent clause.
  17. D. Since both clauses are independent, we must have the correct wording and punctuation to connect them. (A) is incorrect as “on the other hand” does not correctly express the author’s intention. (B) is incorrect since it puts the comma after the fanboy, not before. (C) is incorrect as it connects two independent clauses with a comma. If the comma after “glands” was turned into a semicolon, it would be correct. (D) is the best answer since it ends the first independent clause with a comma, uses correct wording, and has a comma after the introductory work “however”.
  18. B. Answer option (B) is the best answer since it ends the independent clause with a period and has the appropriate comma after the introductory phrase on the second independent clause. (A) is incorrect as it connects two independent clauses with a comma, creating a run-on sentence. (C) is incorrect: while it correctly connects two independent clauses with a semicolon, it lacks the necessary comma after the introductory phrase “in his vision”. (D) is incorrect as it has no punctuation to connect the two independent clauses.
  19. A. In this sentence “we’ve seemingly arbitrarily decided” is extra information. Standard English requires a comma both before and after extra information. Answer option (A) is the only answer option that correctly puts a comma after the word “decided”.
  20. A. In this sentence the phrase “yet no less promising” is extra information, requiring a comma both before and after to set it apart from the rest of the sentence. In addition, there are two independent clauses with the word “and” in between. One option for connecting two independent clauses is a comma along with one of the FANBOYS. We therefore need a comma before the “and”. Option (A) is the only answer that correctly places those three commas within the sentence.

Digital SAT Words in Context Practice Questions

1.  It was comical in a way, with the dramatic irony of a farcical play. He had studied through the lens of his microscope the waste of germs and infinitesimal parasites- invisible to most- that provoke such majorities of the world’s ___________. And yet, in so focusing his view, he had failed to perceive an affliction that towered before him for fifteen years on that to others was unmistakable.

Which of the following choices best completes the text with the most logical word?

A.  Happiness

B.  Suffering

C.  Failure

D.  Calamities

2.  In times of widespread economic distress, experts will sometimes turn to the outliers of a downtrend in order to study the features of their commercial, governmental, and social structures that have seemingly immunized them to financial meltdown. Recently, Sweden’s system has come under particular scrutiny due to the ____________________ of its economy throughout the recession that struck the majority of the Western world in 2007.

Which of the following choices best completes the text with the most precise phrase?

A.  Relatively stable condition

B.  Incredible crash

C.  Loss of the value

D.  Fragmentation

3.  As Mendeleev assembled the table, he had noticed several gaps in the pattern of properties which- cleverly- he hypothesized to be areas held by yet undiscovered elements. Apart from reserving space on the table for these elements, he went to far as to predict not only their existence, but their ______________ as well. Several years later, the spectroscopic discovery of one of these elements- specifically gallium- and the confirmation of Mendeleev’s predictions caused the popularity of his theory to skyrocket, and the periodic table quickly became a standard fixture in the study of chemistry.

Which of the following choices best completes the text with the most logical and precise word or phrase?

A.  Chemical properties

B.  Present locations

C.  Findings

D.  Theory

4.  Once this multitude is united into a body, an offense against one of its members is an offense against the body politic. It would be even less possible to injure the body without its members feeling it. Duty and interest thus equally require the two contracting parties to aid each other ________. The individual people should be motivated from their double roles as individuals and members of the body to combine all the advantages which mutual aid offers them.

Which of the following choices best completes the text with the most logical and precise word?

A.  Completely

B.  Intermittently

C.  Hesitantly

D.  Mutually

5.  The columns, in true Mannerist style, are without function; crowded together chaotically in the corners of the room and crushed halfway into its walls. The pilasters at the sides of the windows are unaccountably tapered- further challenging the neat verticality of classical tradition- and ________ with capitals belonging to no established style whatsoever.

Which of the following choices best completes the text with the most logical and precise word?

A.  Carved

B.  Found

C.  Crowned

D.  Felt

6.  As a whole, the vast exhibition displays how the modernist movement progressed across the three Latin American cultures using furniture as a microcosm of the three societies. Although many might find this to be ________________ of a complex phenomenon, few can argue that the exhibition is, at least, intriguing.

Which of the following choices best completes the text with the most logical and precise phrase?

A.  a misunderstanding

B.  an oversimplification

C.  a failure to display the furniture

D.  an elaboration

7.  Millions of people carry the defective gene, but because it is a recessive disorder, an individual will contract cystic fibrosis only if he or she inherits two mutated genes from his or her parents. If the father and mother both _________ the mutated gene, the child will inherit the disease. However, if only one of the genes is mutated, the child will not contract the disease. If both parents carry the mutated gene, every child they have has a twenty-five percent chance of inheriting the disease and a fifty percent chance of passing it on.

Which of the following choices best completes the text with the most logical and precise word?

A.  lack

B.  grant

C.  avoid

D.  donate

8.  One of the questions that Plato’s fifth book of The Republic grapples with is what role kinship relations play in the function of society. In The Republic, Plato attempts to create a blueprint for a just city-state in which its constituents prescribe to reason and live in communal harmony. In his construction of an ideal city-state, Plato reevaluates the kinship relations that comprise society. Plato believes that the kinship structure is a reflection of the social structure, and that the social and political structures that organize society are not biological or necessary, but rather __________ and subject to change.

Which of the following choices best completes the text with the most logical and precise word?

A.  avoided

B.  unavoidable

C.  exemplary

D.  created

9.  Justice, it might be said, is when tenacity is met with opportunity, and that opportunity came on August 24, 2002 when the Dodgers finally called me up to The Big Show. I pitched that night in a home game against the Braves. The opposing pitcher was Hall of Famer Tom Glavine, and wouldn’t you know that I beat the son of a gun! After the game, my teammates congratulated me in the clubhouse, and, though I don’t remember all of the details, I remember it as one of the ____________ days of my life.

Which of the following choices best completes the text with the most logical and precise word or phrase?

A.  happiest

B.  most stressful

C.  nicest

D.  darkest

10.  Forsythe’s disposition, so recently buoyant and carefree, swiftly darkened to the extent that, were he to auscultate himself at that moment, the _____________ could only reveal an acute saturninity, or else some other sinister malady that corrupts the body and gives the mind a likeness of wetted ash.

Which of the following choices best completes the text with the most logical and precise word?

A.  doctor

B.  patient

C.  feeling

D.  diagnosis

Answer Explanations

1.  B. The character is studying germs and parasites. It is an irony that, though he studies these creatures, he didn’t realize when he himself was “afflicted” with one. It therefore follows that those germs and parasites would create the world’s “suffering” (B). It isn’t (A) as that is the opposite of what the germs and parasites would cause. It isn’t (C) as the world’s failures aren’t caused by germs or parasites.  It isn’t (D) as calamities can be caused by any number of natural disasters and man-made situations in addition to infectious diseases and parasites.

2.  A.   The first sentence of the passage says that during economic disasters, experts turn to outliers. Since the 2007 financial crisis was a recession, the outlier would need to be a country that didn’t experience that recession. Sweden being in a “relatively stable condition” (A) would make it of interest to economists. (B) and (C) are incorrect since if Sweden had crashed or lost value it would not have been an outlier. (D) is incorrect as there is no evidence that the Swedish economy split up into pieces, or fragmented.

3.  D. From the context we know that Mendeleev is predicting the existence of elements as well as something else about them. We know that he reserved spaces for them on the periodic table, so we wouldn’t repeat that he predicted their “present locations” therefore (B) is incorrect. Elements wouldn’t have “findings” or “theory”, so (C) and (D) are incorrect as well. This only leaves “chemical properties,” (A), for him to predict in addition to predicting their existence.

4.  D. In the final sentence we learn that “individuals should be motivated…to combine all the advantages which mutual aid offers them”. This context tells us that the author is calling for the “two contracting parties to aid each other mutually”. In other words, that each should help the other.  This rules out (B) “intermittently” and (C) “hesitantly” since those would both be less than the “mutual aid” the author is calling for. (A) “Completely” is not a word that can be used to describe aiding a person.

5.  C. A “capital” is the top part of a column which often flairs out from the column (or pilar) and is often engraved with a decorative motif. Be careful not to confuse this with a “capitol” which is the building in which a government meets.   Since “capitals” are at the top of a column or pilar, the best word to use in this sentence is (C) “crowned” which indicates that the “capitals” are at the top of the pilar. (A), (B), and (D) would not describe the placement of the capitals.

6.  B. The first sentence sets up a fairly complex topic- the advancement of an entire movement. It makes logical sense that trying to show this complex topic through a display of furniture might be thought to be “an oversimplification” (B) of the topic. There is no evidence that the display is a “misunderstanding” (A) of the movement. We know that the furniture was displayed, so it is not (C). “An elaboration” (D) would be the opposite of an oversimplification and is therefore incorrect.

7. D. In this sentence, the author is describing the process by which both parents must pass on the gene in question in order for the child to have cystic fibrosis. The answer which most closely means “pass on” is “donate”, making (D) the correct answer. If the parents lack or avoid the gene, then the child will not have Cystic Fibrosis, making (A) and (D) incorrect. “Grant” (C) does not fit the context of the sentence.

8.  D. We see in the context of the last sentence that this structure is “not biological or necessary” we are therefore looking for an answer that is the opposite of biological and necessary. “Created” (D) would be the best answer since things that are created can be changed and people can choose not to create them.  They are therefore not “biological” or “necessary”. (A), (B), and (C) do not fit into the context, even though they are grammatically correct.

9.  A. In this passage, the speaker describes the day in which he was given a huge opportunity and not only grabbed that opportunity, but managed to beat one of the best opponents he ever faced. The best way to describe such a day would be as the “happiest” (A) day of his life. “Nicest” (C) is not a strong enough word to describe such a day. “Darkest” (D) is not at all appropriate for such a momentous day.  While the day may have been stressful, the passages is overwhelmingly positive about the experience, so focusing on the negative stress (B) would be inappropriate.

10.  D. While you might not know what “auscultate” means in the passage, context shows that where he to auscultate himself, he was scared of finding a “sinister malady”. This makes “diagnosis” (D) the best option since it is a “diagnosis” that would show such a malady. Since he is auscultating himself, there wouldn’t be a “doctor” involved, so (A) is incorrect. It would be very odd for him to discuss himself in the third person as the “patient”, so (B) is incorrect. He is not looking at a “feeling” but rather at something that will show him something that will “corrupt the body”, making (C) incorrect.